2
$\begingroup$

During my research, I come across this question.

Let $p=3^{101}-4$, $x_0=0$ and $x_{n+1}=x_n^3+1 \mod p$.

Is it true that $\exists k \in \mathbb N \cap [3,p-2], x_k=1$ ?

I have posted this question here : https://artofproblemsolving.com/community/c6h3213569

$\endgroup$

1 Answer 1

7
$\begingroup$

Since $p$ is prime and $\left(\frac{-3}p\right)=-1$, the function $f(x) := x^3 +1$ has the unique inverse and thus represents a permutation on $\mathbb Z/p\mathbb Z$. This means that starting at $x_1=1$ iterative application of $f$ should yield $1$ again. The number of iterations equals the length of the cycle to which $1$ belongs in the permutation $f$.

It remains to notice that $f$ has one fixed point and one cycle of length 3 as established by this PARI/GP code. It follows that the cycle of element 1 has length $\leq p-4$ giving $k$ in the desired interval.

Btw, the inverse of $f$ can be stated explicitly as $f^{-1}(y)=(y-1)^{2\cdot 3^{100}-3}$.

$\endgroup$
13
  • $\begingroup$ So the answer is yes or no ? $\endgroup$ Commented Feb 4, 2024 at 13:28
  • 1
    $\begingroup$ Nice. Btw the same code says that $f^{\circ k}$ has only 1 or 4 fixed points for $k\le 9$, namely 4 iff $3|k$. Thus I don't know if the cycle containing zero has length $p-4$ (which would imply that $f^{\circ (p-3)}(0)=1$). $\endgroup$ Commented Feb 4, 2024 at 14:22
  • 4
    $\begingroup$ @Dattier well, comments is probably not the right place to do so. $\endgroup$ Commented Feb 4, 2024 at 15:00
  • 2
    $\begingroup$ MathOverflow is not intended for challenges. $\endgroup$ Commented Feb 4, 2024 at 21:08
  • 1
    $\begingroup$ @YCor: I've confirmed that there are no other cycles of length $\leq 12$. $\endgroup$ Commented Feb 5, 2024 at 14:07

You must log in to answer this question.

Start asking to get answers

Find the answer to your question by asking.

Ask question

Explore related questions

See similar questions with these tags.